Difference between revisions of "2009 AMC 12B Problems/Problem 19"

Line 78: Line 78:
 
Substituting these values into the second factor and adding would give the answer.
 
Substituting these values into the second factor and adding would give the answer.
  
=== Solution 5 (Using the answer choices) ===
 
Plugging in a few values, it is obvious that <math>f(1) = 41</math> is prime. Starting from <math>n = 2</math>, we know that <math>f(n)</math> becomes negative, then positive again. We want the find the first integral <math>n</math> in which <math>f(n)</math> is positive. This is solved by <math>n^4 -360n^2+400>0</math>.
 
 
 
Instead of using the quadratic formula, we can ignore <math>400</math> to get an approximation. So, we want to solve <math>n^4-360n^2>0</math>.
 
Simplifying, <math>n^2>360</math>. Looking at this, we can automatically see that <math>n = 19</math> is the first value that this happens.
 
 
 
We are only looking for odd values of <math>n</math>. Although we can check if <math>n = 18</math> is positive to account for the <math>400</math> we took off, this will be unnecessary because it wouldn’t be prime anyways.
 
 
 
Plugging in <math>n = 19</math>, we get <math>f(19) = 19^2(19^2-360)+400 = 19^2 \cdot 1+400 = 761</math>, which is prime.
 
 
We can establish that <math>761</math> is prime in two ways. Firstly, we can check prime values until <math>\sqrt{761} \approx 27</math> to see if this is true.
 
 
The second way is faster, but requires the answer choices. We know that <math>41+761 = 802</math> and <math>802</math> is the largest answer choice. Since <math>f(19)</math> is the first odd value of <math>f(n)</math> greater than 0, it follows that <math>f(19)</math> has to be prime for the answer choices to work.
 
 
 
As a result, the answer is <math>\boxed{802}</math>.
 
  
 
== See Also ==
 
== See Also ==

Revision as of 14:40, 15 July 2015

Problem

For each positive integer $n$, let $f(n) = n^4 - 360n^2 + 400$. What is the sum of all values of $f(n)$ that are prime numbers?

$\textbf{(A)}\ 794\qquad \textbf{(B)}\ 796\qquad \textbf{(C)}\ 798\qquad \textbf{(D)}\ 800\qquad \textbf{(E)}\ 802$

Solution

Solution 1

To find the answer it was enough to play around with $f$. One can easily find that $f(1)=41$ is a prime, then $f$ becomes negative for $n$ between $2$ and $18$, and then $f(19)=761$ is again a prime number. And as $41 + 761 = 802$ is already the largest option, the answer must be $\boxed{802}$.

Solution 2

We will now show a complete solution, with a proof that no other values are prime.

Consider the function $g(x) = x^2 - 360x + 400$, then obviously $f(x) = g(x^2)$.

The roots of $g$ are: \[x_{1,2}  = \frac{ 360 \pm \sqrt{ 360^2 - 4\cdot 400 } }2  = 180 \pm 80 \sqrt 5\]

We can then write $g(x) = (x - 180 - 80\sqrt 5)(x - 180 + 80\sqrt 5)$, and thus $f(x) = (x^2 - 180 - 80\sqrt 5)(x^2 - 180 + 80\sqrt 5)$.

We would now like to factor the right hand side further, using the formula $(x^2 - y^2) = (x-y)(x+y)$. To do this, we need to express both constants as squares of some other constants. Luckily, we have a pretty good idea how they look like.

We are looking for rational $a$ and $b$ such that $(a+b\sqrt 5)^2 = 180 + 80\sqrt 5$. Expanding the left hand side and comparing coefficients, we get $ab=40$ and $a^2 + 5b^2 = 180$. We can easily guess (or compute) the solution $a=10$, $b=4$.

Hence $180 + 80\sqrt 5 = (10 + 4\sqrt 5)^2$, and we can easily verify that also $180 - 80\sqrt 5 = (10 - 4\sqrt 5)^2$.

We now know the complete factorization of $f(x)$:

\[f(x) = (x - 10 - 4\sqrt 5)(x + 10 + 4\sqrt 5)(x - 10 + 4\sqrt 5)(x + 10 - 4\sqrt 5)\]

As the final step, we can now combine the factors in a different way, in order to get rid of the square roots.

We have $(x - 10 - 4\sqrt 5)(x - 10 + 4\sqrt 5) = (x-10)^2 - (4\sqrt 5)^2 = x^2 - 20x + 20$, and $(x + 10 - 4\sqrt 5)(x + 10 + 4\sqrt 5) = x^2 + 20x + 20$.

Hence we obtain the factorization $f(x) = (x^2 - 20x + 20)(x^2 + 20x + 20)$.

For $x\geq 20$ both terms are positive and larger than one, hence $f(x)$ is not prime. For $1<x<19$ the second factor is positive and the first one is negative, hence $f(x)$ is not a prime. The remaining cases are $x=1$ and $x=19$. In both cases, $f(x)$ is indeed a prime, and their sum is $f(1) + f(19) = 41 + 761 = \boxed{802}$.

Solution 3

Instead of doing the hard work, we can try to guess the factorization. One good approach:

We can make the observation that $f(x)$ looks similar to $(x^2 + 20)^2$ with the exception of the $x^2$ term. In fact, we have $(x^2 + 20)^2 = x^4 + 40x^2 + 400$. But then we notice that it differs from the desired expression by a square: $f(x) = (x^2 + 20)^2 - 400x^2 = (x^2 + 20)^2 - (20x)^2$.

Now we can use the formula $(x^2 - y^2) = (x-y)(x+y)$ to obtain the same factorization as in the previous solution, without all the work.

Solution 4

After arriving at the factorization $f(x) = (x^2 - 20x + 20)(x^2 + 20x + 20)$, a more mathematical approach would be to realize that the second factor is always positive when $x$ is a positive integer. Therefore, in order for $f(x)$ to be prime, the first factor has to be $1$.

We can set it equal to 1 and solve for $x$:

$x^2-20x+20=1$

$x^2-20x+19=0$

$(x-1)(x-19)=0$

$x=1, x=19$

Substituting these values into the second factor and adding would give the answer.


See Also

2009 AMC 12B (ProblemsAnswer KeyResources)
Preceded by
Problem 18
Followed by
Problem 20
1 2 3 4 5 6 7 8 9 10 11 12 13 14 15 16 17 18 19 20 21 22 23 24 25
All AMC 12 Problems and Solutions

The problems on this page are copyrighted by the Mathematical Association of America's American Mathematics Competitions. AMC logo.png